Convergence des martingales

Bonjour, il est écrit dans mon cours
$X_{n}$ is bounded almost surely so it converges almost surely and in $L_{1}$,
où $X_{n}$ est une martingale. Dois-je comprendre que la convergence se fait dans $L_{1}$ ou que la limite est dans $L_{1}$. Merci pour votre aide.

Bonne journée.

Réponses

  • Les deux !
  • Merci beaucoup poirot ! Maintenant supposons que la suite ne soit plus "bounded a.s" mais "bounded a.s in $L_{1}$" au sens où le sup des espérances de |$X_{n}$| est fini (norme $1$ de la suite majorée indépendamment de $n$).
    Ton affirmation est-elle toujours vraie ?
  • Ça ne veut pas dire grand-chose d'être borné dans $L^1$ presque sûrement, donc tu peux enlever ton "a.s.". Ensuite je ne sais pas répondre à ta question.
  • De quels théorèmes de convergence disposes-tu dans ton cours ?
  • Pardon Poirot je n'ai pas fait attention !

    Siméon :
    pour p =1 : si une martingale est bounded in L1 alors elle converge a.s et la limite est finie
    pour p>1 : si une martingale est bounded in Lp alors elle converge a.s (et aussi dans Lp) et la limite est dans Lp

    where bouded in Lp means sup E|Xn| finite
  • Ok, donc il reste à chercher un exemple de martingale bornée dans $L^1$ mais qui ne converge pas dans $L^1$.

    Faisons simple : on considère la martingale $M$ définie par $n \mapsto X_1X_2\cdots X_n$ où $X$ est une suite i.i.d. de variables de Poisson de paramètre $1$.

    Que vaut $\sup_n E(|M_n|)$ ?
    Quelle est la limite p.s. de cette martingale ?
    A-t-on $\lim_{n\to\infty} E(M_n) = E(\lim_{n\to\infty} M_n)$ ?
  • Le sup vaut 1 donc la martingale est bornée dans L1.

    La limite p.s comme ça je ne vois pas, à moins de dire +infini ce qui est faux.
  • Enfin j'ai envie de dire qu'en prenant un n-uplet, quand n devient infiniment grand alors presque sûrement toutes les variables Xn ne prennent pas la valeur 0 ou 1 et donc presque surement quand n tend vers l'infini Mn=+infini
  • Ok pour le sup.
    Quelle est la probabilité que tous les $X_n$ soient non nuls ?
  • Le produit vaut P(X>0)^n par indépendance c'est-à-dire vaut (1-P(X=0))^n qui vaut 0, ok ?
  • Il n'y a pas de $n$, c'est une variable muette (ici $n\to\infty$), mais ok pour ta conclusion. Peux-tu conclure à partir de là sur la convergence p.s. et la non convergence $L^1$ ?
  • Désolé pour le retard Siméon j'étais absent et merci pour ton aide !

    On a donc une martingale $M_{n}$ qui est bornée dans $L_{1}$. Donc elle converge presque sûrement par le théorème de départ.

    De plus l'espérance de $M_{n}$ vaut $1$ donc à la limite également. Notons $M$ la limite presque sûr de $M_{n}$ est supposons qu'elle soit non nulle presque sûrement. Alors on a en passant à la limite $\lim_{n} P(M_{n}>0) = P(M>0)$ (en passant par les espérances d'indicatrice). Mais grâce à ton aide cela donne $0=1$. C'est absurde et donc $M$ est nulle presque sûrement.

    Finalement $\lim_{n\to\infty} E(M_n) \ne E(\lim_{n\to\infty} M_n)$. Par inégalité triangulaire, si il y avait convergence dans $L_1$ on aurait la convergence vers $0$ de $|E(M_{n})-E(M)|$.
    Ce qui est faux. D'où le contre-exemple.

    C'est tout bon ? (merci encore)
  • Attention au raisonnement : il faudrait justifier qu'ici la limite presque sûre de $M$ est déterministe. En fait, presque sûrement, tous les $M_n$ sont nuls à partir d'un certain rang donc c'est très simple ! Le reste me semble bon.

    P.S. Le bon critère pour la convergence dans $L^1$ est en fait l'uniforme intégrabilité de la martingale.
Connectez-vous ou Inscrivez-vous pour répondre.